Ch 31 Combined

Pataasin ang iyong marka sa homework at exams ngayon gamit ang Quizwiz!

damage to sensory nerve fibers in the arms and legs

peripheral neuropathy

age-related hearing loss

presbycusis

manifesting as farsightedness

presbyopia

innermost layer of the eye

retina

are more sensitive to light and so can provide vision in dim light

rods

two types of photoreceptors in the eyes

rods and cones

has receptor cells that interpret the head's position and maintain equilibrium

semicircular canal

a feeling, within or outside the body, of conditions resulting from stimulation of sensory receptors

sensation

during times of alertnessm, some impulses are ignored by the brain because they are not assigned priority as more important than others

sensory adaptation

decreased stimulation from the environment

sensory deprivation

an overabundance of stimuli

sensory overload

A patient is suspected of having a retinal detachment. What S/S will provide support to this diagnosis? (Select all that apply).

"It feels like I'm looking through cobwebs." "I see specks floating around the edges of my vision." "I feel like someone pulled a curtain over my eye."

A patient is asked to sign a surgical consent for treatment of otosclerosis. Which statement indicates correct understanding of the procedure?

"It will help me hear sounds again."

Yoost: Fundamentals of Nursing, 1st Edition Chapter 31: Cognitive and Sensory Alterations Answer Key for Review Questions (Online-Only) MULTIPLE CHOICE 1. Which area of the brain is involved if a patient is unable to distinguish the shape of an object? Frontal lobes Parietal lobes Temporal lobes Occipital lobes ANS: B The parietal lobes are responsible for the sense of touch, distinguishing the shape and texture of objects. The frontal lobes are responsible for voluntary motor function, concentration, communication, decision making, and personality. The temporal lobes are concerned with hearing and smell. The occipital lobes process visual information. LO: 31.1 DIF: Remembering MSC: NCLEX Client Needs: Physiological Integrity: Physiological Adaption 2. Which patients should be monitored with the Mini Mental State Examination? Patients with cognitive alterations Patients exhibiting signs of depression Patients showing loss of motor function Patients complaining of auditory changes ANS: A The Mini Mental State Examination tests the patient's orientation, language ability, spatial orientation, and attention as well as the ability to calculate and recall. LO: 31.3 DIF: Applying MSC: NCLEX Client Needs: Physiological Integrity: Physiological Adaption 3. What is an appropriate nursing intervention for a patient with expressive aphasia? Direct questions to family members since the patient cannot speak. Make factual statements rather than asking questions. Stand in front of the patient and speak loudly. Use questions with "yes" and "no" answers. ANS: D Patients with expressive aphasia are able to comprehend but cannot express themselves. Use of questions with simple answers helps to involve them in care without increasing frustration. Patients should be included in conversations when the ability to understand is present. Questions and statements are directed to the patient. Speaking loudly will increase the patient's frustration since he or she can hear and understand the spoken word. LO: 31.6 DIF: Understanding MSC: NCLEX Client Needs: Psychosocial Integrity: Sensory/Perceptual Alterations 4. Identify one concern for patients with peripheral neuropathy: burns dizziness confusion visual deficits ANS: A Patients with peripheral neuropathy suffer damage to peripheral nerves, altering the ability to feel extremes of hot and cold. Dizziness can be related to central nervous system disorders or to problems with the inner ear. Confusion is a cognitive deficit. Visual deficits are related to problems with the eye, blood vessels in the eye, or the optic nerve. LO: 31.2 DIF: Remembering MSC: NCLEX Client Needs: Physiological Integrity: Physiological Adaption 5. In a patient with gustatory alterations, which nursing intervention is appropriate? Removal of cerumen Dimming bright lights Turning every two hours Oral hygiene twice daily ANS: D Patients with gustatory alterations have problems with taste. Removal of cerumen improves conductive hearing. Keeping the mouth clean and fresh has a positive impact on taste. Dimming bright lights helps decrease sensory overload. Turning every 2 hours prevents pressure ulcers. LO: 31.6 DIF: Applying MSC: NCLEX Client Needs: Physiological Integrity: Physiological Adaption 6. Which family statement indicates understanding of teaching regarding presbycusis? "I should change positions quickly to stabilize the inner ear." "There are no precautions if I take motion sickness medications." "I should play soft music to distribute vestibular sound." "My family should speak clearly and distinctly." ANS: D The patient with presbycusis has hearing loss and should be spoken to clearly and distinctly. Position changes will not affect the hearing loss. Soft music will not be heard by patients with hearing loss and has no advantages. Patients taking motion sickness medication should refrain from driving. LO: 31.6 DIF: Understanding MSC: NCLEX Client Needs: Psychosocial Integrity: Sensory/Perceptual Alterations 7. Which hospitalized patient is most likely to suffer from sensory deprivation? An ICU patient on a ventilator A bedridden patient with MRSA An ambulatory postoperative patient A patient admitted for diabetes management ANS: B The patient with MRSA would be in contact isolation. Immobility and isolation would prevent this patient from a lot of outside interaction, possibly leading to sensory deprivation. An ICU patient is prone to sensory overload. The ambulatory patient and diabetes patient have the potential to interact with the environment. LO: 31.6 DIF: Understanding MSC: NCLEX Client Needs: Psychosocial Integrity: Sensory/Perceptual Alterations 8. The nurse determines that the patient's self-care abilities have declined. What is the nurse's next step? Re-evaluate the plan of care. Assist the patient with more ADLs. Assess the patient's sensory pathways. Delegate more responsibility to assistive personnel. ANS: A The plan of care is re-evaluated on an ongoing basis. Whenever a patient goal is not met, changes in the plan of care may be necessary. Evaluation takes place before new interventions or delegation occur. LO: 31.6 DIF: Applying MSC: NCLEX Client Needs: Health Promotion and Maintenance: Self Care MULTIPLE RESPONSE 1. Which part of patient care of the cognitively impaired patient can be delegated to assistive personnel? (Select all that apply.) Assistance with hygiene and ambulation Re-orienting the patient to time and place Assessment of cognition and mental status Turning and toileting the patient every two hours Evaluating the effects of nursing interventions on the patient ANS: A, B, D The nurse can delegate routine, repetitive care to assistive personnel. Assessment, planning, and evaluating require critical thinking by the nurse and are therefore not delegated. LO: 31.5 DIF: Understanding MSC: NCLEX Client Needs: Management of Care: Delegation 2. Which nursing interventions should be used if a patient is discovered to be hard of hearing upon initial assessment? (Select all that apply.) Providing written instructions for the patient Standing beside the patient when speaking Enabling closed captioning on the television Speaking into the ear on the side with weaker hearing ANS: A, C Written instructions and closed captioning allow the patient to see words. Standing in from of the patient so lips can be seen is recommended. Speaking in the ear with stronger hearing will increase the likelihood of the patient hearing what is said. LO: 31.6 DIF: Applying MSC: NCLEX Client Needs: Physiological Integrity: Physiological Adaptation

...

In planning for the discharge of a client with a cognitive disorder, it is important to assess the client's caregiver support system. Which aspects are the most crucial to assess? Select all that apply. 1. Availability of resources for caregiver support. 2. Ability to provide the level of care and supervision needed by the client. 3. Willingness to transport the client to medical and psychiatric services. 4. Interest in engaging the cognitively disordered family member in reminiscence and games. 5. Willingness to install door alarms and make other safety changes. 6. Understanding the client's abilities and limitations.

1, 2, 3, 5, 6. It is important for a caregiver to have support for herself as well as be able to provide adequate safety, supervision, and medical care to the client. The caregiver must also have realistic expectations of the client, given his abilities and limitations. Reminiscing and engaging the client in games is desirable but not crucial to care.

An 83-year-old woman is admitted to the unit after being examined in the emergency department (ED) and diagnosed with delirium. After the admission interviews with the client and her grandson, the nurse explains that there will be more laboratory tests and X-rays done that day. The grandson says, "She has already been stuck several times and had a brain scan or something. Just give her some medicine and let her rest." The nurse should tell the grandson which of the following? Select all that apply. 1. "I agree she needs to rest, but there is no one specific medicine for your grandmother's condition." 2. "The doctor will look at the results of those tests in the ED and decide what other tests are needed." 3. "Delirium commonly results from underlying medical causes that we need to identify and correct." 4. "Tell me about your grandmother's behaviors and maybe I could figure out what medicine she needs." 5. "I'll ask the doctor to postpone more tests until tomorrow."

1, 2, 3. The client does need rest and it is true that there is no specific medicine for delirium, but it is crucial to identify and treat the underlying causes of delirium. Other tests will be based on the results of already completed tests. Although some medications may be prescribed to help the client with her behaviors, this is not the primary basis for medication orders. Because the underlying medical causes of delirium could be fatal, treatment must be initiated as soon as possible. It is not the nurse's role to determine medications for this client. Postponing tests until the next day is inappropriate.

A client has been in the critical care unit for 3 days following a severe myocardial infarction. Although he is medically stable, he has begun to have fluctuating episodes of consciousness, illogical thinking, and anxiety. He is picking at the air to "catch these baby angels flying around my head." While waiting for medical and psychiatric consults, the nurse must intervene with the client's needs. Which of the following needs have the highest priority? Select all that apply. 1. Decreasing as much "foreign" stimuli as possible. 2. Avoiding challenging the client's perceptions about "baby angels." 3. Orienting the client about his medical condition. 4. Gently presenting reality as needed. 5. Calling the client's family to report his onset of dementia.

1, 2, 4. The abnormal stimuli of the critical care unit can aggravate the symptoms of delirium. Arguing with hallucinations is inappropriate. When a client has illogical thinking, gently presenting reality is appropriate. Dementia is not the likely cause of the client's symptoms. The client is experiencing delirium, not dementia.

Transfer data for a client brought by ambulance to the hospital's psychiatric unit from a nursing home indicate that the client has become increasingly confused and disoriented. The client's behavior is found to be the result of cerebral arteriosclerosis. Which of the following behaviors of the nursing staff should positively influence the client's behavior? Select all that apply. 1. Limiting the client's choices. 2. Accepting the client as he is. 3. Allowing the client to do as he wishes. 4. Acting nonchalantly. 5. Explaining to the client what he needs to do step-by-step.

1, 2, 5. Confused clients need fewer choices, acceptance as a person, and step-by-step directions. Allowing the client to do as he wishes can lead to substandard care and the risk of harm. Acting nonchalantly conveys a lack of caring.

The nurse is making a home visit with a client diagnosed with Alzheimer's disease. The client recently started on lorazepam (Ativan) due to increased anxiety. The nurse is cautioning the family about the use of lorazepam (Ativan). The nurse should instruct the family to report which of the following significant side effects to the health care provider? 1. Paradoxical excitement. 2. Headache. 3. Slowing of reflexes. 4. Fatigue.

1. Although all of the side effects listed are possible with Ativan, paradoxical excitement is cause for immediate discontinuation of the medication. (Paradoxical excitement is the opposite reaction to Ativan than is expected.) The other side effects tend to be minor and usually are transient.

The client with dementia states to the nurse, "I know you. You're Margaret, the girl who lives down the street from me." Which of the following responses by the nurse is most therapeutic? 1. "Mrs. Jones, I'm Rachel, a nurse here at the hospital." 2. "Now Mrs. Jones, you know who I am." 3. "Mrs. Jones, I told you already, I'm Rachel and I don't live down the street." 4. "I think you forgot that I'm Rachel, Mrs. Jones."

1. Because of the client's short-term memory impairment, the nurse gently corrects the client by stating her name and who she is. This approach decreases anxiety, embarrassment, and shame and maintains the client's self-esteem. Telling the client that she knows who the nurse is or that she forgot can elicit feelings of embarrassment and shame. Saying, "I told you already" sounds condescending, as if blaming the client for not remembering.

When developing the plan of care for a client with Alzheimer's disease who is experiencing moderate impairment, which of the following types of care should the nurse expect to include? 1. Prompting and guiding activities of daily living. 2. Managing a medication schedule. 3. Constant supervision and total care. 4. Supervision of risky activities such as shaving.

1. Considerable assistance is associated with moderate impairment when the client cannot make decisions but can follow directions. Managing medications is needed even in mild impairment. Constant care is needed in the terminal phase, when the client cannot follow directions. Supervision of shaving is appropriate with mild impairment— that is, when the client still has motor function but lacks judgment about safety issues.

The nurse is attempting to draw blood from a woman with a diagnosis of delirium who was admitted last evening. The client yells out, "Stop; leave me alone. What are you trying to do to me? What's happening to me?" Which response by the nurse is most appropriate? 1. "The tests of your blood will help us figure out what is happening to you." 2. "Please hold still so I don't have to stick you a second time." 3. "After I get your blood, I'll get some medicine to help you calm down." 4. "I'll tell you everything after I get your blood tests to the laboratory."

1. Explaining why blood is being taken responds to the client's concerns or fears about what is happening to her. Threatening more pain or promising to explain later ignores or postpones meeting the client's need for information. The client's statements do not reflect loss of self control requiring medication intervention.

In addition to developing over a period of hours or days, the nurse should assess delirium as distinguishable by which of the following characteristics? 1. Disturbances in cognition and consciousness that fluctuate during the day. 2. The failure to identify objects despite intact sensory functions. 3. Significant impairment in social or occupational functioning over time. 4. Memory impairment to the degree of being called amnesia.

1. Fluctuating symptoms are characteristic of delirium. The failure to identify objects despite intact sensory functions, significant impairment in social or occupational functioning over time, and memory impairment to the degree

Which of the following should the nurse expect to include as a priority in the plan of care for a client with delirium based on the nurse's understanding about the disturbances in orientation associated with this disorder? 1. Identifying self and making sure that the nurse has the client's attention. 2. Eliminating the client's napping in the daytime as much as possible. 3. Engaging the client in reminiscing with relatives or visitors. 4. Avoiding arguing with a suspicious client about his perceptions of reality.

1. Identifying oneself and making sure that the nurse has the client's attention addresses the difficulties with focusing, orientation, and maintaining attention. Eliminating daytime napping is unrealistic until the cause of the delirium is determined and the client's ability to focus and maintain attention improves. Engaging the client in reminiscing and avoiding arguing are also unrealistic at this time.

The client in the early stage of Alzheimer's disease and his adult son attend an appointment at the community mental health center. While conversing with the nurse, the son states, "I'm tired of hearing about how things were 30 years ago. Why does Dad always talk about the past?" The nurse should tell the son: 1. "Your dad lost his short-term memory, but he still has his long-term memory." 2. "You need to be more accepting of your dad's behavior." 3. "I want you to understand your dad's level of anxiety." 4. "Telling your dad that you are tired of hearing about the past will help him stop."

1. The son's statements regarding his father's recalling past events is typical for family members of clients in the early stage of Alzheimer's disease, when recent memory is impaired. Telling the son to be more accepting is critical and not an attempt to educate. Understanding the client's level of anxiety is unrelated to the memory loss of Alzheimer's disease. The client cannot stop reminiscing at will.

When helping the families of clients with Alzheimer's disease cope with vulgar or sexual behaviors, which of the following suggestions is most helpful? 1. Ignore the behaviors, but try to identify the underlying need for the behaviors. 2. Give feedback on the inappropriateness of the behaviors. 3. Employ anger management strategies. 4. Administer the prescribed risperidone (Risperdal).

1. The vulgar or sexual behaviors are commonly expressions of anger or more sensual needs that can be addressed directly. Therefore, the families should be encouraged to ignore the behaviors but attempt to identify their purpose. Then the purpose can be addressed, possibly leading to a decrease in the behaviors. Because of impaired cognitive function, the client is not likely to be able to process the inappropriateness of the behaviors if given feedback. Likewise, anger management strategies would be ineffective because the client would probably be unable to process the inappropriateness of the behaviors. Risperidone (Risperdal) may decrease agitation, but it does not improve social behaviors.

Nursing staff are trying to provide for the safety of an elderly female client with moderate dementia. She is wandering at night and has trouble keeping her balance. She has fallen twice but has had no resulting injuries. The nurse should: 1. Move the client to a room near the nurse's station and install a bed alarm. 2. Have the client sleep in a reclining chair across from the nurse's station. 3. Help the client to bed and raise all four bedrails. 4. Ask a family member to stay with the client at night.

1. Using a bed alarm enables the staff to respond immediately if the client tries to get out of bed. Sleeping in a chair at the nurse's station interferes with the client's restful sleep and privacy. Using all four bedrails is considered a restraint and unsafe practice. It is not appropriate to expect a family member to stay all night with the client.

The husband of a client with Alzheimer's disease that was diagnosed 6 years ago approaches the nurse and says, "I'm so excited that my wife is starting to use donepezil (Aricept) for her illness." The nurse should tell the husband: 1. The medication is effective mostly in the early stages of the illness. 2. The adverse effects of the drug are numerous. 3. The client will attain a functional level of that of 6 years ago. 4. Effectiveness in the terminal phase of the illness is scientifically proven.

1. When compared with other similar medications, donepezil (Aricept) has fewer adverse effects. Donepezil is effective primarily in the early stages of the disease. The drug helps to slow the progression of the disease if started in the early stages. After the client has been diagnosed for 6 years, improvement to the level seen 6 years ago is highly unlikely. Data are not available to support the drug's effectiveness for clients in the terminal phase of the disease.

During a home visit to an elderly client with mild dementia, the client's daughter reports that she has one major problem with her mother. She says, "She sleeps most of the day and is up most of the night. I can't get a decent night's sleep anymore." Which suggestions should the nurse make to the daughter? Select all that apply. 1. Ask the client's physician for a strong sleep medicine. 2. Establish a set routine for rising, hygiene, meals, short rest periods, and bedtime. 3. Engage the client in simple, brief exercises or a short walk when she gets drowsy during the day. 4. Promote relaxation before bedtime with a warm bath or relaxing music. 5. Have the daughter encourage the use of caffeinated beverages during the day to keep her mother awake.

2, 3, 4. A set routine and brief exercises help decrease daytime sleeping. Decreasing caffeine and fluids and promoting relaxation at bedtime promote nighttime sleeping. A strong sleep medicine for an elderly client is contraindicated due to changes in metabolism, increased adverse effects, and the risk of falls. Using caffeinated beverages may stimulate metabolism but can also have long-lasting adverse effects and may prevent sleep at bedtime.

The family of a client, diagnosed with Alzheimer's disease, wants to keep the client at home. They say that they have the most difficulty in managing his wandering. The nurse should instruct the family to do which of the following? (Select all that apply). 1. Ask the physician for a sleeping medication. 2. Install motion and sound detectors. 3. Have a relative sit with the client all night. 4. Have the client wear a Medical Alert bracelet. 5. Install door alarms and high door locks.

2, 4, 5. Motion and sound detectors, a Medical Alert bracelet, and door alarms are all appropriate interventions for wandering. Sleep medications do not prevent wandering before and after the client is asleep and may have negative effects. Having a relative sit with the client is usually an unrealistic burden.

Which of the following is a priority to include in the plan of care for a client with Alzheimer's disease who is experiencing difficulty processing and completing complex tasks? 1. Repeating the directions until the client follows them. 2. Asking the client to do one step of the task at a time. 3. Demonstrating for the client how to do the task. 4. Maintaining routine and structure for the client.

2. Because the client is experiencing difficulty processing and completing complex tasks, the priority is to provide the client with only one step at a time, thereby breaking the task up into simple steps, ones that the client can process. Repeating the directions until the client follows them or demonstrating how to do the task is still too overwhelming to the client because of the multiple steps involved. Although maintaining structure and routine is important, it is unrelated to task completion.

A client with early dementia exhibits disturbances in her mental awareness and orientation to reality. The nurse should expect to assess a loss of ability in which of the following other areas? 1. Speech. 2. Judgment. 3. Endurance. 4. Balance.

2. Clients with chronic cognitive disorders experience defects in memory orientation and intellectual functions, such as judgment and discrimination. Loss of other abilities, such as speech, endurance, and balance, is less typical.

A client diagnosed with dementia wanders the halls of the locked nursing unit during the day. To ensure the client's safety while walking in the halls, the nurse should do which of the following? 1. Administer PRN haloperidol (Haldol) to decrease the need to walk. 2. Assess the client's gait for steadiness. 3. Restrain the client in a geriatric chair. 4. Administer PRN lorazepam (Ativan) to provide sedation.

2. Elderly clients have increased risk for falls due to balance problems, medication use, and decreased eyesight. Haldol may cause extrapyramidal side effects (EPSE) which increase the risk for falls. The client is not agitated, so restraints are not indicated. Ativan may increase fall risk and cause paradoxical excitement.

The term motor apraxia relates to a decline in motor patterns essential for complex motor tasks. However, the client with severe dementia may be able to perform which of the following actions? 1. Balance a checkbook accurately. 2. Brush the teeth when handed a toothbrush. 3. Use confabulation when telling a story. 4. Find misplaced car keys.

2. Highly conditioned motor skills, such as brushing the teeth, may be retained by the client who has dementia and motor apraxia. Balancing a checkbook involves calculations, a complex skill that is lost with severe dementia Confabulation is fabrication of details to fill a memory gap. This is more common when the client is aware of a memory problem, not when dementia is severe. Finding keys is a memory factor, not a motor function.

A 69-year-old client is admitted and diagnosed with delirium. Later in the day, he tries to get out of the locked unit. He yells, "Unlock this door. I've got to go see my doctor. I just can't miss my monthly Friday appointment." Which of the following responses by the nurse is most appropriate? 1. "Please come away from the door. I'll show you your room." 2. "It's Tuesday and you are in the hospital. I'm Anne, a nurse." 3. "The door is locked to keep you from getting lost." 4. "I want you to come eat your lunch before you go the doctor."

2. Loss of orientation, especially for time and place, is common in delirium. The nurse should orient the client by telling him the time, date, place, and who the client is with. Taking the client to his room and telling him why the door is locked does not address his disorientation. Telling the client to eat before going to the doctor reinforces his disorientation.

Which of the following is essential when caring for a client who is experiencing delirium? 1. Controlling behavioral symptoms with low-dose psychotropics. 2. Identifying the underlying causative condition or illness. 3. Manipulating the environment to increase orientation. 4. Decreasing or discontinuing all previously prescribed medications.

2. The most critical aspect when caring for the client with delirium is to institute measures to correct the underlying causative condition or illness. Controlling behavioral symptoms with low-dose psychotropics, manipulating the environment, and decreasing or discontinuing all medications may be dangerous to the client's health.

A nurse on the Geropsychiatric unit receives a call from the son of a recently discharged client. He reports that his father just got a prescription for memantine (Namenda) to take "on top of his donepezil (Aricept)." The son then asks, "Why does he have to take extra medicines?" The nurse should tell the son: 1. "Maybe the Aricept alone isn't improving his dementia fast enough or well enough." 2. "Namenda and Aricept are commonly used together to slow the progression of dementia." 3. "Namenda is more effective than Aricept. Your father will be tapered off the Aricept." 4. "Aricept has a short half-life and Namenda has a long half-life. They work well together."

2. The two medicines are commonly given together. Neither medicine will improve dementia, but may slow the progression. Neither medicine is more effective than the other; they act differently in the brain. Both medicines have a half-life of 60 or more hours.

While assessing a client diagnosed with dementia, the nurse notes that her husband is concerned about what he should do when she uses vulgar language with him. The nurse should: 1. Tell her that she is very rude. 2. Ignore the vulgarity and distract her. 3. Tell her to stop swearing immediately. 4. Say nothing and leave the room.

2. Vulgar language is common in clients with dementia when they are having trouble communicating about a topic. Ignoring the vulgarity and distracting her is appropriate. Telling the client she is rude or to stop swearing will have no lasting effect and may cause agitation. Just leaving the room is abandonment that the client will not understand.

A client is experiencing agnosia as a result of vascular dementia. She is staring at dinner and utensils without trying to eat. Which intervention should the nurse attempt first? 1. Pick up the fork and feed the client slowly. 2. Say, "It's time for you to start eating your dinner." 3. Hand the fork to the client and say, "Use this fork to eat your green beans." 4. Save the client's dinner until her family comes in to feed her.

3. Agnosia is the lack of recognition of objects and their purpose. The nurse should inform the client about the fork and what to do with it. Feeding the client does not address the agnosia or give the client specific directions. It should only be attempted if identifying the fork and explaining what to do with it is ineffective. Waiting for the family to care for the client is not appropriate unless identifying the fork and explaining or feeding the client are not successful.

The physician orders risperidone (Risperdal) for a client with Alzheimer's disease. The nurse anticipates administering this medication to help decrease which of the following behaviors? 1. Sleep disturbances. 2. Concomitant depression. 3. Agitation and assaultiveness. 4. Confusion and withdrawal.

3. Antipsychotics are most effective with agitation and assaultiveness. Antipsychotics have little effect on sleep disturbances, concomitant depression, or confusion and withdrawal.

When communicating with the client who is experiencing dementia and exhibiting decreased attention and increased confusion, which of the following interventions should the nurse employ as the first step? 1. Using gentle touch to convey empathy. 2. Rephrasing questions the client doesn't understand. 3. Eliminating distracting stimuli such as turning off the television. 4. Asking the client to go for a walk while talking.

3. Competing and excessive stimuli lead to sensory overload and confusion. Therefore, the nurse should first eliminate any distracting stimuli. After this is accomplished, then using touch and rephrasing questions are appropriate. Going for a walk while talking has little benefit on attention and confusion.

Which of the following is a realistic short-term goal to be accomplished in 2 to 3 days for a client with delirium? 1. Explain the experience of having delirium. 2. Resume a normal sleep-wake cycle. 3. Regain orientation to time and place. 4. Establish normal bowel and bladder function.

3. In approximately 2 to 3 days, the client should be able to regain orientation and thus become oriented to time and place. Being able to explain the experience of having delirium is something that the client is expected to achieve later in the course of the illness, but ultimately before discharge. Resuming a normal sleep-wake cycle and establishing normal bowel and bladder function probably will take longer, depending on how long it takes to resolve the underlying condition.

When caring for the client diagnosed with delirium, which condition is the most important for the nurse to investigate? 1. Cancer of any kind. 2. Impaired hearing. 3. Prescription drug intoxication. 4. Heart failure.

3. Polypharmacy is much more common in the elderly. Drug interactions increase the incidence of intoxication from prescribed medications, especially with combinations of analgesics, digoxin, diuretics, and anticholinergics. With drug intoxication, the onset of the delirium typically is quick. Although cancer, impaired hearing, and heart failure could lead to delirium in the elderly, the onset would be more gradual.

The nurse observes a client in a group who is reminiscing about his past. Which effect should the nurse expect reminiscing to have on the client's functioning in the hospital? 1. Increase the client's confusion and disorientation. 2. Cause the client to become sad. 3. Decrease the client's feelings of isolation and loneliness. 4. Keep the client from participating in therapeutic activities.

3. Reminiscing can help reduce depression in an elderly client and lessens feelings of isolation and loneliness. Reminiscing encourages a focus on positive memories and accomplishments as well as shared memories with other clients. An increase in confusion and disorientation is most likely the result of other cognitive and situational factors, such as loss of short-term memory, not reminiscing. The client will not likely become sad because reminiscing helps the client connect with positive memories. Keeping the client from participating in therapeutic activities is less likely with reminiscing.

The nurse discusses the possibility of a client's attending day treatment for clients with early Alzheimer's disease. Which of the following is the best rationale for encouraging day treatment? 1. The client would have more structure to his day. 2. Staff are excellent in the treatment they offer clients. 3. The client would benefit from increased social interaction. 4. The family would have more time to engage in their daily activities.

3. The best rationale for day treatment for the client with Alzheimer's disease is the enhancement of social interactions. More daily structure, excellent staff, and allowing caregivers more time for themselves are all positive aspects, but they are less focused on the client's needs.

The client with Alzheimer's disease may have delusions about being harmed by staff and others. When the client expresses fear of being killed by staff, which of the following responses is most appropriate? 1. "What makes you think we want to kill you?" 2. "We like you too much to want to kill you." 3. "You are in the hospital. We are nurses trying to help you." 4. "Oh, don't be so silly. No one wants to kill you here."

3. The nurse needs to present reality without arguing with the delusions. Therefore, stating that the client is in the hospital and the nurses are trying to help is most appropriate. The client doesn't recognize the delusion or why it exists. Telling the client that the staff likes him too much to want to kill him is inappropriate because the client believes the delusions and doesn't know that they are false beliefs. It also restates the word, kill, which may reinforce the client's delusions. Telling the client not to be silly is condescending and disparaging and therefore inappropriate.

A client with dementia who prefers to stay in his room has been brought to the dayroom. After 10 minutes, the client becomes agitated and retreats to his room again. The nurse decides to assess the conditions in the dayroom. Which is the most likely occurrence that is disturbing to this client? 1. There is only one other client in the dayroom; the rest are in a group session in another room. 2. There are three staff members and one physician in the nurse's station working on charting. 3. A relaxation tape is playing in one corner of the room, and a television airing a special on crime is playing in the opposite corner. 4. A housekeeping staff member is washing off the countertops in the kitchen, which is on the far side of the dayroom.

3. The tape and television are competing, even conflicting, stimuli. Crime events portrayed on television could be misperceived as a real threat to the client. A low number of clients and the presence of a few staff members quietly working are less intense stimuli for the client and not likely to be disturbing.

When providing family education for those who have a relative with Alzheimer's disease about minimizing stress, which of the following suggestions is most relevant? 1. Allow the client to go to bed four to five times during the day. 2. Test the cognitive functioning of the client several times a day. 3. Provide reality orientation even if the memory loss is severe. 4. Maintain consistency in environment, routine, and caregivers.

4. Change increases stress. Therefore, the most important and relevant suggestion is to maintain consistency in the client's environment, routine, and caregivers. Although rest periods are important, going to bed interferes with the sleep-wake cycle. Rest in a recliner chair is more useful. Testing cognitive functioning and reality orientation are not likely to be successful and may increase stress if memory loss is severe.

An elderly woman's husband died. When her brother arrives for the funeral, he notices her short-term memory problems and occasional disorientation. A few weeks later, she calls him to say that her husband just died. She says, "I didn't know he was so sick. Why did he die now?" She also complains of not sleeping, urinary frequency and burning, and seeing rats in the kitchen. A home care nurse is sent to evaluate her situation and finds the woman reclusive and passive, but pleasant. The nurse calls the woman's primary care physician to discuss the client's situation and background, and give his assessment and recommendations. The nurse concludes that the woman: 1. Is experiencing the onset of Alzheimer's disease. 2. Is having trouble adjusting to living alone without her husband. 3. Is having delayed grieving related to her Alzheimer's disease. 4. Is experiencing delirium and a urinary tract infection.

4. Delirium is commonly due to a medical condition such as a UTI in the elderly. Delirium often involves memory problems, disorientation, and hallucinations. It develops rather quickly. There is not enough data to suggest Alzheimer's disease especially given the quick onset of symptoms. Delayed grieving and adjusting to being alone are unlikely to cause hallucinations.

The son of an elderly client who has cognitive impairments approaches the nurse and says, "I'm so upset. The physician says I have 4 days to decide on where my dad is going to live." The nurse responds to the son's concerns, gives him a list of types of living arrangements, and discusses the needs, abilities, and limitations of the client. The nurse should intervene further if the son makes which comment? 1. "Boy, I have a lot to think about before I see the social worker tomorrow." 2. "I think I can handle most of Dad's needs with the help of some home health care." 3. "I'm so afraid of making the wrong decision, but I can move him later if I need to." 4. "I want the social worker to make this decision so Dad won't blame me."

4. Expecting the social worker to make the decision indicates that the son is avoiding participating in decisions about his father. The other responses convey that the son understands the importance of a careful decision, the availability of resources, and the ability to make new plans if needed.

While educating the daughter of a client with dementia about the illness, the daughter complains to the nurse that her mother distorts things. The nurse understands that the daughter needs further teaching about dementia when she makes which statement? 1. "I tell her reality, such as, 'That noise is the wind in the trees.'" 2. "I understand the misperceptions are part of the disease." 3. "I turn off the radio when we're in another room." 4. "I tell her she is wrong and then I tell her what's right."

4. Telling the client that she is wrong and then telling her what is right is argumentative and challenging. Arguing with or challenging distortions is least effective because it increases defensiveness. Telling the client about reality indicates awareness of the issues and is appropriate. Acknowledging that misperceptions are part of the disease indicates an understanding of the disease and an awareness of the issues. Turning off the radio helps to limit environmental stimuli and indicates an awareness of the issues.

A 90-year-old client diagnosed with major depression is suddenly experiencing sleep disturbances, inability to focus, poor recent memory, altered perceptions, and disorientation to time and place. Lab results indicate the client has a urinary tract infection and dehydration. After explaining the situation and giving the background and assessment data, the nurse should make which of the following recommendations to the client's physician? 1. An order to place the client in restraints. 2. A reevaluation of the client's mental status. 3. The transfer of the client to a medical unit. 4. A transfer of the client to a nursing home.

4. The client is showing symptoms of delirium, a common outcome of UTI in older adults. The nurse can request a transfer to a medical unit for acute medical intervention. The client's symptoms are not just due to a worsening of the depression. There are not indications that the client needs restraints or a transfer to a nursing home at this point.

The nurse determines that the son of a client with Alzheimer's disease needs further education about the disease when he makes which of the following statements? 1. "I didn't realize the deterioration would be so incapacitating." 2. "The Alzheimer's support group has so much good information." 3. "I get tired of the same old stories, but I know it's important for Dad." 4. "I woke up this morning expecting that my old Dad would be back."

4. The statement about expecting that the old Dad would be back conveys a lack of acceptance of the irreversible nature of the disease. The statement about not realizing that the deterioration would be so incapacitating is based in reality. The statement about the Alzheimer's group is based in reality and demonstrates the son's involvement with managing the disease. Stating that reminiscing is important reflects a realistic interpretation on the son's part.

The nurse is assigned to care for a client with a detached retina. Which finding should the nurse expect to be documented in the client's record?

A sense of a curtain falling across the field of vision.

The nurse notes that the health care provider has documented a diagnosis of presbycusis on the client's chart. The nurse understands that this condition is accurately described as which?

A sensorineural hearing loss that occurs with aging.

The patient reports a loss of central vision. What test should the nurse teach the patient about to identify changes in macular function? A) Amsler grid test B) B-scan ultrasonography C) Fluorescein angiography D) Intraocular pressure testing with Tono-Pen

A) Amsler grid test Rationale: The Amsler grid test is self-administered and regular testing is necessary to identify any changes in macular function. B-scan ultrasonography is used to diagnose ocular pathologic conditions (e.g., intraocular foreign bodies or tumors, vitreous opacities, retinal detachments). Fluorescein angiography is used to diagnose problems related to the flow of blood through pigment epithelial and retinal vessels. Intraocular pressure testing with a Tono-Pen is done to test for glaucoma.

A college student reports eye pain after studying for finals. What assessment should the nurse make first in determining the possible etiology of this eye pain? A) Do you wear contacts? B) Do you have any allergies? C) Do you have double vision? D) Describe the change in your vision.

A) Do you wear contacts? Rationale: College students frequently wear contact lenses and stay up late or all night studying for finals. If the student wears contacts, the wearing of them while studying, care of them, and length of wear time will be assessed before looking for a corneal abrasion from extended wear with fluorescein dye. There are no manifestations of allergies, diplopia, or visual changes mentioned.

A patient reporting frequent vertigo is scheduled for electronystagmography to test vestibular function. Which instructions should the nurse provide to the patient before the procedure? A) Eat a light meal before the procedure. B) Avoid carbonated beverages before the procedure. C) Take nothing by mouth for 3 hours before the procedure. D) No special dietary restrictions are needed until after the procedure.

A) Eat a light meal before the procedure. Rationale: Instruct patient to eat a light meal before the test to avoid nausea. Results of vestibular tests can be altered by use of caffeine, other stimulants, sedatives, and antivertigo drugs.

An older adult patient states they don't seem to hear well and have to ask people to repeat themselves. What should the nurse do first to determine the cause of the hearing loss? A) Look for cerumen in the ear. B) Assess for increased hair growth in the ear. C) Tell the patient it is probably related to aging. D) Ask the patient if he has fallen because of dizziness.

A) Look for cerumen in the ear. Rationale: Gerontologic differences in the assessment of the auditory system include increased production of and drier cerumen, which can become impacted in the ear canal and contribute to hearing loss. Conductive hearing loss with impacted cerumen may lead to speaking softly as the patient's voice conducted through bone seems loud to the patient. Although increased hair growth occurs, it will not impact the hearing. Presbycusis may be occurring, but it should not be assumed. There is no reason to ask the patient if he has fallen because dizziness and vertigo are not a normal change of aging of the ear.

Identify one concern for patients with peripheral neuropathy: a. burns b. dizziness c. confusion d. visual deficits

ANS: A Patients with peripheral neuropathy suffer damage to peripheral nerves, altering the ability to feel extremes of hot and cold. Dizziness can be related to central nervous system disorders or to problems with the inner ear. Confusion is a cognitive deficit. Visual deficits are related to problems with the eye, blood vessels in the eye, or the optic nerve.

Which patients should be monitored with the Mini Mental State Examination? a. Patients with cognitive alterations b. Patients exhibiting signs of depression c. Patients showing loss of motor function d. Patients complaining of auditory changes

ANS: A The Mini Mental State Examination tests the patient's orientation, language ability, spatial orientation, and attention as well as the ability to calculate and recall.

The nurse determines that the patient's self-care abilities have declined. What is the nurse's next step? a. Re-evaluate the plan of care. b. Assist the patient with more ADLs. c. Assess the patient's sensory pathways. d. Delegate more responsibility to assistive personnel.

ANS: A The plan of care is re-evaluated on an ongoing basis. Whenever a patient goal is not met, changes in the plan of care may be necessary. Evaluation takes place before new interventions or delegation occur.

Identify one concern for patients with peripheral neuropathy: a. burns b. dizziness c. confusion d. visual deficits

ANS: A Patients with peripheral neuropathy suffer damage to peripheral nerves, altering the ability to feel extremes of hot and cold. Dizziness can be related to central nervous system disorders or to problems with the inner ear. Confusion is a cognitive deficit. Visual deficits are related to problems with the eye, blood vessels in the eye, or the optic nerve.

Which patients should be monitored with the Mini Mental State Examination? a. Patients with cognitive alterations b. Patients exhibiting signs of depression c. Patients showing loss of motor function d. Patients complaining of auditory changes

ANS: A The Mini Mental State Examination tests the patient's orientation, language ability, spatial orientation, and attention as well as the ability to calculate and recall.

The nurse determines that the patient's self-care abilities have declined. What is the nurse's next step? a. Re-evaluate the plan of care. b. Assist the patient with more ADLs. c. Assess the patient's sensory pathways. d. Delegate more responsibility to assistive personnel.

ANS: A The plan of care is re-evaluated on an ongoing basis. Whenever a patient goal is not met, changes in the plan of care may be necessary. Evaluation takes place before new interventions or delegation occur.

Which part of patient care of the cognitively impaired patient can be delegated to assistive personnel? (Select all that apply.) a. Assistance with hygiene and ambulation b. Re-orienting the patient to time and place c. Assessment of cognition and mental status d. Turning and toileting the patient every two hours e. Evaluating the effects of nursing interventions on the patient

ANS: A, B, D The nurse can delegate routine, repetitive care to assistive personnel. Assessment, planning, and evaluating require critical thinking by the nurse and are therefore not delegated.

Which part of patient care of the cognitively impaired patient can be delegated to assistive personnel? (Select all that apply.) a. Assistance with hygiene and ambulation b. Re-orienting the patient to time and place c. Assessment of cognition and mental status d. Turning and toileting the patient every two hours e. Evaluating the effects of nursing interventions on the patient

ANS: A, B, D The nurse can delegate routine, repetitive care to assistive personnel. Assessment, planning, and evaluating require critical thinking by the nurse and are therefore not delegated.

Which nursing interventions should be used if a patient is discovered to be hard of hearing upon initial assessment? (Select all that apply.) a. Providing written instructions for the patient b. Standing beside the patient when speaking c. Enabling closed captioning on the television d. Speaking into the ear on the side with weaker hearing

ANS: A, C Written instructions and closed captioning allow the patient to see words. Standing in from of the patient so lips can be seen is recommended. Speaking in the ear with stronger hearing will increase the likelihood of the patient hearing what is said.

Which nursing interventions should be used if a patient is discovered to be hard of hearing upon initial assessment? (Select all that apply.) a. Providing written instructions for the patient b. Standing beside the patient when speaking c. Enabling closed captioning on the television d. Speaking into the ear on the side with weaker hearing

ANS: A, C Written instructions and closed captioning allow the patient to see words. Standing in from of the patient so lips can be seen is recommended. Speaking in the ear with stronger hearing will increase the likelihood of the patient hearing what is said.

Which area of the brain is involved if a patient is unable to distinguish the shape of an object? a. Frontal lobes b. Parietal lobes c. Temporal lobes d. Occipital lobes

ANS: B The parietal lobes are responsible for the sense of touch, distinguishing the shape and texture of objects. The frontal lobes are responsible for voluntary motor function, concentration, communication, decision making, and personality. The temporal lobes are concerned with hearing and smell. The occipital lobes process visual information.

Which hospitalized patient is most likely to suffer from sensory deprivation? a. An ICU patient on a ventilator b. A bedridden patient with MRSA c. An ambulatory postoperative patient d. A patient admitted for diabetes management

ANS: B The patient with MRSA would be in contact isolation. Immobility and isolation would prevent this patient from a lot of outside interaction, possibly leading to sensory deprivation. An ICU patient is prone to sensory overload. The ambulatory patient and diabetes patient have the potential to interact with the environment.

Which area of the brain is involved if a patient is unable to distinguish the shape of an object? a. Frontal lobes b. Parietal lobes c. Temporal lobes d. Occipital lobes

ANS: B The parietal lobes are responsible for the sense of touch, distinguishing the shape and texture of objects. The frontal lobes are responsible for voluntary motor function, concentration, communication, decision making, and personality. The temporal lobes are concerned with hearing and smell. The occipital lobes process visual information.

Which hospitalized patient is most likely to suffer from sensory deprivation? a. An ICU patient on a ventilator b. A bedridden patient with MRSA c. An ambulatory postoperative patient d. A patient admitted for diabetes management

ANS: B The patient with MRSA would be in contact isolation. Immobility and isolation would prevent this patient from a lot of outside interaction, possibly leading to sensory deprivation. An ICU patient is prone to sensory overload. The ambulatory patient and diabetes patient have the potential to interact with the environment.

3. What is an appropriate nursing intervention for a patient with expressive aphasia? a. Direct questions to family members since the patient cannot speak. b. Make factual statements rather than asking questions. c. Stand in front of the patient and speak loudly. d. Use questions with "yes" and "no" answers.

ANS: D Patients with expressive aphasia are able to comprehend but cannot express themselves. Use of questions with simple answers helps to involve them in care without increasing frustration. Patients should be included in conversations when the ability to understand is present. Questions and statements are directed to the patient. Speaking loudly will increase the patient's frustration since he or she can hear and understand the spoken word.

In a patient with gustatory alterations, which nursing intervention is appropriate? a. Removal of cerumen b. Dimming bright lights c. Turning every two hours d. Oral hygiene twice daily

ANS: D Patients with gustatory alterations have problems with taste. Removal of cerumen improves conductive hearing. Keeping the mouth clean and fresh has a positive impact on taste. Dimming bright lights helps decrease sensory overload. Turning every 2 hours prevents pressure ulcers.

Which family statement indicates understanding of teaching regarding presbycusis? a. "I should change positions quickly to stabilize the inner ear." b. "There are no precautions if I take motion sickness medications." c. "I should play soft music to distribute vestibular sound." d. "My family should speak clearly and distinctly."

ANS: D The patient with presbycusis has hearing loss and should be spoken to clearly and distinctly. Position changes will not affect the hearing loss. Soft music will not be heard by patients with hearing loss and has no advantages. Patients taking motion sickness medication should refrain from driving.

In a patient with gustatory alterations, which nursing intervention is appropriate? a. Removal of cerumen b. Dimming bright lights c. Turning every two hours d. Oral hygiene twice daily

ANS: D Patients with gustatory alterations have problems with taste. Removal of cerumen improves conductive hearing. Keeping the mouth clean and fresh has a positive impact on taste. Dimming bright lights helps decrease sensory overload. Turning every 2 hours prevents pressure ulcers.

What is an appropriate nursing intervention for a patient with expressive aphasia? a. Direct questions to family members since the patient cannot speak. b. Make factual statements rather than asking questions. c. Stand in front of the patient and speak loudly. d. Use questions with "yes" and "no" answers.

ANS: D Patients with expressive aphasia are able to comprehend but cannot express themselves. Use of questions with simple answers helps to involve them in care without increasing frustration. Patients should be included in conversations when the ability to understand is present. Questions and statements are directed to the patient. Speaking loudly will increase the patient's frustration since he or she can hear and understand the spoken word.

Which family statement indicates understanding of teaching regarding presbycusis? a. "I should change positions quickly to stabilize the inner ear." b. "There are no precautions if I take motion sickness medications." c. "I should play soft music to distribute vestibular sound." d. "My family should speak clearly and distinctly."

ANS: D The patient with presbycusis has hearing loss and should be spoken to clearly and distinctly. Position changes will not affect the hearing loss. Soft music will not be heard by patients with hearing loss and has no advantages. Patients taking motion sickness medication should refrain from driving.

The nurse is evaluating a patient's eye as it adjust to seeing objects a various distances. When documenting, how should the nurse identify this test?

Accommodation.

A patient who has been blind for the past 10 years is hospitalized with heart failure. What intervention should the nurse include in the plan of care?

All personnel announce themselves when entering and leaving the room.

most common type of dementia

Alzheimer's disease

protin fragments called amyloid plaques build up between the nerve cells of the brain, blocking electrical and chemical connections between neurons

Alzheimer's disease

When caring for an elderly patient who presents with acute confusion of sudden onset, which test would the nurse expect to be ordered? a. Urine culture and sensitivity testing b. Mini-Mental State Examination c. Swallow evaluation d. MRI with contrast

Answer: a A major cause of acute confusion in the elderly is infections including urinary tract infections and pneumonia. Urine culture and sensitivity testing will detect bacteria in the urine and determine proper antibiotic treatment. A Mini-Mental State Exam is a valuable tool to assess the progression of dementia. Swallow evaluation is done in patients who are suspected of having a weak or absent gag reflex. MRI with contrast might be done in a patient with confusion after infection has been ruled out.

Which nursing intervention is appropriate for a patient with sensory overload? a. Dimming the lights b. Performing care a little at a time c. Leaving the patient's door open d. Rushing to get care done quickly

Answer: a Dimming the lights decreases sensory stimuli which alleviates sensory overload. Constant disruption adds to the overload, as does leaving the door open and rushing while in the room. A calm, quiet atmosphere diminishes the overload.

When caring for a hearing impaired patient, use of which technique by the nurse would facilitate communication? a. Speaking clearly with distinct words b. Talking slowly to facilitate understanding c. Sitting behind the patient to decrease distractions d. Standing near the patient's affected ear to balance sound

Answer: a Speaking clearly without shouting facilitates communication with the hearing-impaired patient by giving each word separate emphasis. Talking distinctly, but not too slowly, and allowing the patient to see facial expressions and read lips, with the use of hearing aids if prescribed, are good communication techniques. Speaking into ear with the better hearing is recommended.

Which statement by the patient with vertigo lets the nurse know that the patient has understood the home-going instructions? a. "I will buy a visual signal for my smoke detectors." b. "I will have grab bars installed in my bathtub." c. "I will change positions quickly to avoid vertigo." d. "I will get a home phone with amplified sound."

Answer: b Grab bars provide stability for the patient with vertigo. Patients with vertigo should change positions slowly to avoid worsening of the spinning sensation. Visual signals and amplified sound are used in the home of the patient with hearing deficits.

A visually impaired diabetic patient states that he has lost the call light. What is the next step the nurse should take? a. Clip the call light closer to the patient. b. Tell the patient that the call light is clipped to the bed. c. Describe the call light location, and take the patient's hand and guide it to that location. d. Instruct the patient to verbally call for a staff member because "someone is always nearby."

Answer: c Always leave the call light within easy reach of the patient. Use of the patient's senses of both touch and hearing enables the patient to locate the call light easier. Simply telling the patient that the call light is clipped to the bed is not adequate, because the patient will not know where on the bed to look. Verbally calling for the nurse is not acceptable because the nurse and other staff members might be out of hearing range.

Which goal statement is appropriate for a patient with the nursing diagnosis of Impaired Memory? a. Patient will remember nurse's name. b. Nurse will remind patient of his or her name each shift. c. Patient will state name and date with each nursing encounter. d. Nurse will remind patient of name and date with each nursing encounter.

Answer: c Goals are always patient-centered and measurable and have a specified time frame. A patient goal would not include a nursing behavior. A confused patient would not be expected to remember different nurses' names but would be assessed for person, place, and time orientation with each encounter.

Which nursing diagnosis is most appropriate for a patient with presbycusis? a. Impaired Verbal Communication b. Disturbed Thought Processes c. Disturbed Sensory Perception d. Impaired Physical Mobility

Answer: c Presbycusis is age-related hearing loss. The nursing diagnosis of Disturbed Sensory Perception is appropriate for patients with visual, auditory, kinesthetic, gustatory, tactile, or olfactory deficits. Impaired Verbal Communication relates to the ability to speak and form words. Disturbed Thought Processes is appropriate for patients with cognitive deficits. Patients with limitations of movement may have a nursing diagnosis of Impaired Physical Mobility.

The nurse is caring for a patient with decreased sensation in the lower extremities. Which precaution does the nurse advise the patient to take? a. Use heat to warm hands during cold weather. b. Go barefoot at home to prevent blisters from shoes. c. Soak feet in cold water daily to decrease swelling. d. Test the bath water temperature to prevent burning injuries.

Answer: d Because the patient may not be able to feel the temperature of the water, using a thermometer will prevent burns. The use of heat and cold is contraindicated in patients with tactile deficits because they would not be able to feel if the therapy was too hot or cold. The patient should wear good-fitting shoes around the house to prevent foot injury.

Which recommendation in the home-going instructions is appropriate for a patient with damage to the chemoreceptors of the upper nasal passages? a. Arranging for lighted signals on doorbells and telephones b. Obtaining a thermometer for testing bath water temperature c. Installing amplification devices on televisions, doorbells, and telephones d. Scheduling yearly safety checks of gas, hot water heaters, and furnaces

Answer: d Patients with damage to the chemoreceptors of the nasal passages may not be able to smell noxious fumes from household appliances. Lighted signals and amplification are interventions for a person with auditory deficits. Testing the bath water temperature is important for patients with tactile deficits.

Which nursing interventions would be necessary in caring for a patient with cognitive alterations who is hospitalized? (Select all that apply.) a. Apply wrist restraints for combativeness. b. Place a clock in the room for orientation. c. Keep floor free of clutter for safety. d. Identify staff with each interaction. e. Play loud music for distraction.

Answers: b, c, d Reality orientation is important for patients with cognitive alterations. Keeping the floor free of clutter prevents falls. All staff members should wear a readily visible name tag and state their name and what they are going to do. Soft music and dim lights will create a less distracting environment for the patient. Restraints may cause increased confusion and agitation and are used in special circumstances only.

A client sustains a contusion of the eyeball after a traumatic injury with a blunt object. The nurse should take which immediate action?

Apply ice to the affected eye.

Which of the following would be most hazardous in the home of a patient who is visually impaired?

Area rugs.

A client with Meniere's disease is experiencing severe vertigo. The nurse reinforces instructions to the client to do which to assist in controlling vertigo?

Avoid sudden head movements.

A 71-year-old patient complains of being severely dizzy. What instruction should the nurse give the patient?

Avoid sudden movements.

A patient complains of intermittent eye dryness. Which question should the nurse ask the patient to determine the etiology of this symptom? A) "Do you take ginkgo to treat asthma or tinnitus?" B) "What do you take if you have allergy symptoms?" C) "Are you taking propranolol for an anxiety disorder?" D) "How long have you been taking prednisone (Deltasone)?"

B) "What do you take if you have allergy symptoms?" Rationale: Antihistamines or decongestants taken for allergies or colds can cause ocular dryness. Ginkgo biloba is an herbal product and has been used to treat asthma and tinnitus. Side effects of ginkgo may include headache, nausea, gastrointestinal upset, diarrhea, dizziness, allergic skin reactions, and increased bleeding. β-Adrenergic blockers can potentiate drugs used to treat glaucoma. Long-term use of prednisone (corticosteroids) may contribute to the development of glaucoma or cataracts.

A patient has ptosis resulting from myasthenia gravis. Which assessment finding would the nurse expect to see in this patient? A) Redness and swelling of the conjunctiva B) Drooping of the upper lid margin in one or both eyes C) Redness, swelling, and crusting along the eyelid margins D) Small, superficial white nodules along the eyelid margin

B) Drooping of the upper lid margin in one or both eyes Rationale: Ptosis is the term used to describe drooping of the upper eyelid margin, which may be either unilateral or bilateral. Ptosis can be a result of mechanical causes, such as an eyelid tumor or excess skin, or from myogenic causes such as myasthenia gravis. Redness, swelling of the conjunctive, or crusting along the eyelid margins may indicate an infection such as viral or bacterial conjunctivitis. Small superficial white nodules along the eyelid margin may indicate hordeolum (sty).

The nurse is assessing a patient's medical history. What aspects of the patient's medical history are most likely to have potential consequences for the patient's visual system? A) Hypothyroidism and polycythemia B) Hypertension and diabetes mellitus C) Atrial fibrillation and atherosclerosis D) Vascular dementia and chronic fatigue

B) Hypertension and diabetes mellitus Rationale: Hypertension and diabetes frequently contribute to visual pathologies. The other health problems are less likely to have a direct, deleterious effect on a patient's vision.

When examining the patient's ear with an otoscope, the nurse observes discharge in the canal and the patient reports pain with the examination. What should the nurse next assess the patient for? A) Sebaceous cyst B) Swimmer's ear C) Metabolic disorder D) Serous otitis media

B) Swimmer's ear Rationale: Swimmer's ear or an infection of the external ear is probably the cause of the discharge and pain. Asking the patient about swimming, ear protection, and exposure to types of water can identify contact with contaminated water. After clearing the discharge, the tympanic membrane can be assessed for otitis media. A sebaceous cyst and metabolic disorders would not cause drainage or discomfort in the external ear canal.

A patient has a family history of cataracts. He asks what early symptoms he should watch for that would alert him to the development of cataracts. What is the nurse's best response?

Blurred vision.

A patient working in a noisy factory reports being off balance when standing or walking but not while lying down. What term will the nurse use to document this patient's symptoms? A) Vertigo Incorrect B) Syncope C) Dizziness Correct D) Nystagmus

C) Dizziness Rationale: Dizziness is a sensation of being off balance that occurs when standing or walking; it does not occur when lying down. Nystagmus is an abnormal eye movement that may be observed as a twitching of the eyeball or described by the patient as a blurring of vision with head or eye movement. Vertigo is a sense that the person or objects around the person are moving or spinning and is usually stimulated by movement of the head. Syncope is a brief lapse in consciousness accompanied by a loss in postural tone (fainting).

When assessing an adult patient's external ear canal and tympanum, what assessment techniques should the nurse use? A) Ask the patient to tip his or her head toward the nurse. B) Identify a pearl gray tympanic membrane as a sign of infection. C) Gently pull the auricle up and backward to straighten the canal. D) Identify a normal light reflex by the appearance of irregular edges.

C) Gently pull the auricle up and backward to straighten the canal. Rationale: When examining a patient's external ear canal and tympanum, ask the patient to tilt the head toward the opposite shoulder. Grasp and gently pull the auricle up and backward to straighten the canal. A healthy, normal tympanic membrane will appear pearl gray, white, or pink and have a cone-shaped light reflex.

A patient newly diagnosed with glaucoma asks the nurse what has made the pressure in the eyes so high. Which is the nurse's most accurate response? A) Back pressure from cardiac congestion causes corneal edema. B) Cerebral venous dilation prevents normal interstitial fluid resorption. C) Increased production of aqueous humor or blocked drainage increases pressure. Correct D) Congenital anomalies of the lacrimal gland or duct obstruct the passage of tears.

C) Increased production of aqueous humor or blocked drainage increases pressure. Rationale: Intraocular pressure is increased in glaucoma as a result of excess aqueous humor production or decreased outflow. Cardiac or cerebral circulation changes do not cause glaucoma. Lacrimal anomalies do not affect aqueous humor production.

Otoscopic examination of the patient's left ear indicates the presence of an exostosis. What does the nurse prepare to teach the patient about regarding the growth? A) Surgery B) Electrocochleography C) Monitoring of the growth D) Irrigation of the ear canal

C) Monitoring of the growth Rationale: An exostosis is a bony growth into the ear canal that normally does not require intervention or correction.

During a health history, a 43-yr-old teacher complains of increasing difficulty reading printed materials for the past year. What change related to aging does the nurse suspect? A) Myopia Incorrect B) Hyperopia C) Presbyopia Correct D) Astigmatism

C) Presbyopia Rationale: Presbyopia is a loss of accommodation causing an inability to focus on near objects. This occurs as a normal part of aging process starting around age 40 years. Myopia is nearsightedness (near objects are clear and far objects are blurred). Astigmatism results in visual distortion related to unevenness in the cornea. Hyperopia is farsightedness (near objects are blurred and far objects are clearly seen).

The nurse is admitting an adult patient to a walk-in clinic. The patient complains of recent hearing loss. What does the nurse anticipate as the most probable cause of this patient's hearing loss?

Cerumen buildup

After cataract surgery a patient complains of sudden sharp pain in the operative eye. What is the most appropriate nursing action?

Call the surgeon.

A client is diagnosed with glaucoma. Which data gathered by the nurse indicated a risk factor associated with glaucoma?

Cardiovascular disease.

The nurse is assigned to care for a client with a diagnosis of detached retina. Which finding would indicate that bleeding has occurred as a result of retinal detachment?

Complaints of burst of black spots or floaters.

A patient visits the health care provider to have her vision tested using the Snellen eye chart. What instruction should the nurse provide to the patient?

Cover one eye while testing the other.

The nurse is assessing an older adult patient who has just been transferred to the long-term care facility. Which assessment question will best allow the nurse to assess for the presence of presbycusis? A) "Do you ever experience any ringing in your ears?" B) "Have you ever fallen down because you became dizzy?" C) "Do you ever have pain in your ears when you're chewing or swallowing?" D) "Have you noticed any change in your hearing in recent months and years?"

D) "Have you noticed any change in your hearing in recent months and years?" Rationale: Presbycusis is an age-related change in auditory acuity. Whereas ringing in the ears is termed tinnitus, dizziness and falls are related to balance and the function of the vestibular system. Presbycusis is not associated with pain during chewing and swallowing.

During the course of an interview to assess vision, a patient complains of dry eyes. What should the nurse implement next? A) Assess for contact lenses. B) Suggest saline eye drops. C) Ask about eyeglass usage. D) Check the medication list.

D) Check the medication list. Rationale: The nurse should evaluate the patient's medication list to identify agents that can contribute to dry eyes so follow-up nursing care can be planned. Dry eyes aggravate wearing contact lenses, but contact lenses do not normally cause dry eyes. The nurse should not suggest saline eye drops until the etiology of the dry eyes is determined. Eyeglasses do not cause dry eyes.

The nurse is assisting in developing a teaching plan for the client with glaucoma. Which instruction should the nurse suggest to include in the plan of care?

Eye medications will need to be administered for the rest of your life.

A patient has impaired hearing. Which action by the nurse would best facilitate communication?

Face the patient when speaking.

A 15-year-old hearing-impaired patient is having problems communicating with the staff. Which behavior would improve communication? (Select all that apply.)

Facing the patient when speaking. Speaking in conversational tones. Asking permission to turn off the television or radio.

Which assessment finding would indicate a need for possible glaucoma testing? (Select all that apply.)

Halos around lights. Progressive loss of peripheral vision. Lack of ability to adapt to darkness.

A client arrives at the emergency department with a foreign body in the left ear that has been determined to be an insect. Which initial intervention should the nurse anticipate to be prescribed?

Instillation of mineral oil or diluted alcohol

While cleaning the garage a patient splashed a chemical in his eyes. What is the initial action following the chemical burn?

Irrigate the eye with water for 20 minutes.

A client sustains a chemical eye injury from a splash of battery acid. The nurse should prepare the client for which immediate measure?

Irrigating the eye with sterile normal saline.

A patient arrives in the emergency room following an accident that resulted in a piece of metal penetrating the eye. What nursing action should be taken initially on the patient's arrival at the hospital?

Lightly cover the eyes with an eye shield.

The nurse is assigned to care for a client hospitalized with Meniere's disease. The nurse expects that which would most likely be prescribed for the client?

Low-sodium diet.

A 78-year-old patient comes into the clinic complaining of progressive loss of vision in the center of the visual field. The nurse is aware that the patient is most likely experiencing symptoms of which disorder?

Macular degeneration.

The nurse receives medication orders for a patient with open-angle glaucoma. Which medication order does the nurse anticipate?

Mannitol, Cyclogyl

A client arrives in the emergency department after an automobile crash. The client's forehead hit the steering wheel, and hyphema has been diagnosed. Which position should the nurse prepare to position the client?

On bed rest in a semi-Fowler's position.

A patient is to have a laser treatment to cauterize hemorrhaging vessels caused by diabetic retinopathy. The patient asks the nurse what this procedure is called. Which response by the nurse is correct?

Photocoagulation.

The nurse is preparing to administer eardrops to an adult client. The nurse administers the eardrops by which technique?

Pulling the pinna UP and BACK

The nurse is caring for a patient with vertigo. What is the nurse's priority concern when caring for this patient?

Safety.

The parents want to know more about their child's conductive hearing loss. Which is the best explanation by the nurse?

Sound is inadequately delivered through the external or middle ear to the inner ear.

While communication with a patient, you notice a possible hearing deficit in one ear. Which nursing intervention would be appropriate?

Speak clearly and in a slightly louder voice toward the patient's face.

The nurse is caring for a client who is hearing impaired and should take which approach to facilitate communication?

Speak in a normal tone.

A patient is scheduled for a stapedectomy. What postoperative instructions should the nurse include in patient teaching? (Select all that apply.)

Teach the patient to open the mouth when sneezing or coughing. Limit exercise or active sports for 3 weeks. Avoid exposure to people with upper respiratory tract infections.

The nurse is preparing to reinforce a teaching plan for a client who is undergoing cataract extraction with intraocular implant. Which home care measures should the nurse include in the plan? SELECT ALL THAT APPLY.

To avoid activities that require bending over. To place an eye shield on the surgical eye at bedtime. To contact the surgeon if a decrease in visual acuity occurs. To take acetaminophen (Tylenol) for minor eye discomfort.

A patient is diagnosed with an inner ear problem. What symptom should the nurse monitor the patient closely for?

Vertigo

A patient tells the nurse he has dizziness. He states that the health care provider used another term. What term did the health care provider most likely use?

Vertigo.

A patient has just had cataract surgery. What information should the nurse include in the discharge instructions? (Select all the apply.)

Wear an eye shield at night on the operative eye. Avoid bending, stooping, coughing, or lifting. Instill prescribed eyedrops into the conjunctival sac.

complete loss of the sense of smell

anosmia

a decrease in size

atrophy

outer ear

auricle

clouding of the lens of the eye

cataract

occurs when an area of the brain is deprived of blood flow

cerebrovascular accident (CVA) Stroke

sensory nerve endings that react to chemicals

chemoreceptors

knowing influenced by awareness and judgement, it is composed of skills that include language, calculation, memory, attention, reasoning, learning, problem solving, and decision making

cognition

detect sharp, color images

cones

sense of smell

olfaction

three small bones in the middle ear

ossicles

impulses cross over

decussate

a reversible state of acute confusion

delirium

permanent decline in mental function, has a subtle onset

dementia

mood disorder characterized by a sense of hopelessness and persitent unhappiness

depression

complication of diabetes mellitus in which the blood vessels of the retina becomes damaged

diabetic retinopathy

state of balance

equilibrium

causes increased intraocular pressure, which puts pressure on the optic nerve, leading to loss of peripheral visual fields and possibly blindness

glaucoma

sense of taste

gustation

the way the brain perceives the information

perception

Yoost: Fundamentals of Nursing Chapter 31: Cognitive and Sensory Alterations Answer Key for Review Questions 1. A visually impaired diabetic patient states that he has lost the call light. What is the next step the nurse should take? a. Clip the call light closer to the patient. b. Tell the patient that the call light is clipped to the bed. c. Describe the call light location, and take the patient's hand and guide it to that location. d. Instruct the patient to verbally call for a staff member because "someone is always nearby." Answer: c Always leave the call light within easy reach of the patient. Use of the patient's senses of both touch and hearing enables the patient to locate the call light easier. Simply telling the patient that the call light is clipped to the bed is not adequate, because the patient will not know where on the bed to look. Verbally calling for the nurse is not acceptable because the nurse and other staff members might be out of hearing range. LO: 31.6 Blooms: Applying TOP: Visual Impairment MSC: NCLEX Client Needs: Psychosocial integrity: Sensory/perceptual alterations 2. When caring for a hearing impaired patient, use of which technique by the nurse would facilitate communication? a. Speaking clearly with distinct words b. Talking slowly to facilitate understanding c. Sitting behind the patient to decrease distractions d. Standing near the patient's affected ear to balance sound Answer: a Speaking clearly without shouting facilitates communication with the hearing-impaired patient by giving each word separate emphasis. Talking distinctly, but not too slowly, and allowing the patient to see facial expressions and read lips, with the use of hearing aids if prescribed, are good communication techniques. Speaking into ear with the better hearing is recommended. LO: 31.6 Blooms: Applying TOP: Hearing Impairment MSC: NCLEX Client Needs: Psychosocial integrity: Sensory/perceptual alterations 3. The nurse is caring for a patient with decreased sensation in the lower extremities. Which precaution does the nurse advise the patient to take? a. Use heat to warm hands during cold weather. b. Go barefoot at home to prevent blisters from shoes. c. Soak feet in cold water daily to decrease swelling. d. Test the bath water temperature to prevent burning injuries. Answer: d Because the patient may not be able to feel the temperature of the water, using a thermometer will prevent burns. The use of heat and cold is contraindicated in patients with tactile deficits because they would not be able to feel if the therapy was too hot or cold. The patient should wear good-fitting shoes around the house to prevent foot injury. LO: 31.6 Blooms: Applying TOP: Tactile Deficits MSC: NCLEX Client Needs: Safe and effective care environment: Home safety 4. Which statement by the patient with vertigo lets the nurse know that the patient has understood the home-going instructions? a. "I will buy a visual signal for my smoke detectors." b. "I will have grab bars installed in my bathtub." c. "I will change positions quickly to avoid vertigo." d. "I will get a home phone with amplified sound." Answer: b Grab bars provide stability for the patient with vertigo. Patients with vertigo should change positions slowly to avoid worsening of the spinning sensation. Visual signals and amplified sound are used in the home of the patient with hearing deficits. LO: 31.6 Blooms: Applying TOP: Equilibrium Alterations MSC: NCLEX Client Needs: Safe and effective care environment: Home safety 5. Which nursing intervention is appropriate for a patient with sensory overload? a. Dimming the lights b. Performing care a little at a time c. Leaving the patient's door open d. Rushing to get care done quickly Answer: a Dimming the lights decreases sensory stimuli which alleviates sensory overload. Constant disruption adds to the overload, as does leaving the door open and rushing while in the room. A calm, quiet atmosphere diminishes the overload. LO: 31.6 Blooms: Applying TOP: Sensory Overload MSC: NCLEX Client Needs: Psychosocial integrity: Sensory/perceptual alterations 6. Which recommendation in the home-going instructions is appropriate for a patient with damage to the chemoreceptors of the upper nasal passages? a. Arranging for lighted signals on doorbells and telephones b. Obtaining a thermometer for testing bath water temperature c. Installing amplification devices on televisions, doorbells, and telephones d. Scheduling yearly safety checks of gas, hot water heaters, and furnaces Answer: d Patients with damage to the chemoreceptors of the nasal passages may not be able to smell noxious fumes from household appliances. Lighted signals and amplification are interventions for a person with auditory deficits. Testing the bath water temperature is important for patients with tactile deficits. LO: 31.6 Blooms: Applying TOP: Olfactory Alterations MSC: NCLEX Client Needs: Safe and effective care environment: Home safety 7. When caring for an elderly patient who presents with acute confusion of sudden onset, which test would the nurse expect to be ordered? a. Urine culture and sensitivity testing b. Mini-Mental State Examination c. Swallow evaluation d. MRI with contrast Answer: a A major cause of acute confusion in the elderly is infections including urinary tract infections and pneumonia. Urine culture and sensitivity testing will detect bacteria in the urine and determine proper antibiotic treatment. A Mini-Mental State Exam is a valuable tool to assess the progression of dementia. Swallow evaluation is done in patients who are suspected of having a weak or absent gag reflex. MRI with contrast might be done in a patient with confusion after infection has been ruled out. LO: 31.3 Blooms: Analyzing TOP: Delirium MSC: NCLEX Client Needs: Safe and effective care environment: Safety and infection control 8. Which nursing diagnosis is most appropriate for a patient with presbycusis? a. Impaired Verbal Communication b. Disturbed Thought Processes c. Disturbed Sensory Perception d. Impaired Physical Mobility Answer: c Presbycusis is age-related hearing loss. The nursing diagnosis of Disturbed Sensory Perception is appropriate for patients with visual, auditory, kinesthetic, gustatory, tactile, or olfactory deficits. Impaired Verbal Communication relates to the ability to speak and form words. Disturbed Thought Processes is appropriate for patients with cognitive deficits. Patients with limitations of movement may have a nursing diagnosis of Impaired Physical Mobility. LO: 31.4 Blooms: Applying TOP: Nursing Process: Diagnosis MSC: NCLEX Client Needs: Psychosocial integrity: Sensory/perceptual alterations 9. Which nursing interventions would be necessary in caring for a patient with cognitive alterations who is hospitalized? (Select all that apply.) a. Apply wrist restraints for combativeness. b. Place a clock in the room for orientation. c. Keep floor free of clutter for safety. d. Identify staff with each interaction. e. Play loud music for distraction. Answers: b, c, d Reality orientation is important for patients with cognitive alterations. Keeping the floor free of clutter prevents falls. All staff members should wear a readily visible name tag and state their name and what they are going to do. Soft music and dim lights will create a less distracting environment for the patient. Restraints may cause increased confusion and agitation and are used in special circumstances only. LO: 31.6 Blooms: Applying TOP: Nursing Process: Planning MSC: NCLEX Client Needs: Psychosocial integrity: Sensory/perceptual alterations 10. Which goal statement is appropriate for a patient with the nursing diagnosis of Impaired Memory? a. Patient will remember nurse's name. b. Nurse will remind patient of his or her name each shift. c. Patient will state name and date with each nursing encounter. d. Nurse will remind patient of name and date with each nursing encounter. Answer: c Goals are always patient-centered and measurable and have a specified time frame. A patient goal would not include a nursing behavior. A confused patient would not be expected to remember different nurses' names but would be assessed for person, place, and time orientation with each encounter. LO: 31.5 Blooms: Applying TOP: Nursing Process: Planning MSC: NCLEX Client Needs: Psychosocial integrity: Sensory/perceptual alterations

jhvjuvuj

intricate communicating passageways

labyrinths

The leading cause of visual defects in the US Typically begins after the age of 50. Loss of vision occurs in the central visual fields Causes include diabetes, genetics, smoking, and hypertension

macular degeneration

nearsightedness

myopia

twisted fragments of protein that clog the nerve cell

neurofibrillary tangles

a change in the environment sufficient to evoke a response,occurs, sensory receptors trigger a response, which results in nerve impulses ascending sensory pathways to the central nervous system for processing

stimulus

worsening of agitation and confusion in the evening

sundowning

senses detectable by touch

tactile

a ringing or other abnormal sound in the ear

tinnitus

sensation that objects are moving around the person

vertigo

Chapter 31: Cognitive and Sensory Alterations MULTIPLE CHOICE 1. A nurse is caring for a patient with a stroke that has impacted her ability to see. Which area of the brain was likely impacted by the stroke that is responsible for visual function? a. Parietal lobes b. Frontal lobes c. Occipital lobes d. Temporal lobes ANS: C The occipital lobes process visual information. The frontal lobes of the cerebrum are the areas of the brain responsible for voluntary motor function, concentration, communication, decision making, and personality. The parietal lobes are responsible for the sense of touch, distinguishing the shape and texture of objects. The temporal lobes are concerned with the senses of hearing and smell. DIF: Applying REF: p. 710 OBJ: 31.1 TOP: Implementation MSC: NCLEX Client Needs Category: Physiological Integrity NOT: Concepts: Cognition 2. The family of a patient who was in a motor vehicle accident tells you he "just isn't the same person before the crash." You know this is likely because of the injury to what area of his brain? a. Parietal lobes b. Frontal lobes c. Occipital lobes d. Temporal lobes ANS: B The frontal lobes of the cerebrum are the areas of the brain responsible for voluntary motor function, concentration, communication, decision making, and personality. The parietal lobes are responsible for the sense of touch, distinguishing the shape and texture of objects. The temporal lobes are concerned with the senses of hearing and smell. The occipital lobes process visual information. DIF: Applying REF: p. 710 OBJ: 31.1 TOP: Implementation MSC: NCLEX Client Needs Category: Physiological Integrity NOT: Concepts: Cognition 3. The nurse is educating the family of a patient in the intensive care unit about the patient's cognitive status, including her current problem of delirium. Which statement by the family indicates a need for further education? a. "The delirium can be caused by sensory overload." b. "The delirium is reversible." c. "The delirium is a mood disorder." d. "The delirium is a state of confusion." ANS: C Delirium is a reversible state of acute confusion. It is characterized by a disturbance in consciousness or a change in cognition that develops over 1 to 2 days and is caused by a medical condition. Delirium may occur in intensive care patients as a result of sensory overload. It is not a mood disorder. DIF: Applying REF: p. 713 OBJ: 31.2 TOP: Implementation MSC: NCLEX Client Needs Category: Physiological Integrity NOT: Concepts: Cognition 4. The nurse is caring for a patient with depression. Which statement by the patient indicates a need for further education? a. "Depression can be caused by chemical changes in the brain." b. "Depression is always treated with medication." c. "Depression is a mood disorder." d. "Depression can have a rapid onset." ANS: B Depression is usually reversible with treatment either by eliminating the underlying cause, providing counseling, or prescribing antidepressive agents. Depression is a mood disorder and is believed to be caused by chemical changes in the brain. Depression usually has a rapid onset, and the patient's mood is constant. DIF: Applying REF: p. 713 OBJ: 31.2 TOP: Implementation MSC: NCLEX Client Needs Category: Physiological Integrity NOT: Concepts: Cognition 5. The nurse is caring for a patient who is complaining of tingling in her hands and fingers. The nurse knows this is a sign of what electrolyte imbalance? a. Hyponatremia b. Hypernatremia c. Hypocalcemia d. Hypercalcemia ANS: C Tactile disturbances, such as tingling and numbness around the mouth and in the fingers, are signs of hypocalcemia. Mental changes are associated with both hypercalcemia and hypocalcemia. Both hypernatremia and hyponatremia have symptoms of central nervous system disorder. DIF: Applying REF: p. 717 OBJ: 31.3 TOP: Implementation MSC: NCLEX Client Needs Category: Physiological Integrity NOT: Concepts: Cognition 6. The nurse is providing discharge instructions to an older adult who is being discharged with orthostatic hypotension. Which of the following responses by the patient indicates a need for further education? a. "I should take my blood pressure once a day at home." b. "I should get up quickly to avoid my blood pressure dropping." c. "I should drink plenty of water during the day." d. "I should get up slowly and carefully." ANS: B In orthostatic hypotension, dizziness and loss of consciousness may occur if a patient changes position too quickly. Instead they should change positions slowly. A patient can take their blood pressure at home to monitor it, and drinking water will keep them hydrated. DIF: Applying REF: p. 717 OBJ: 31.3 TOP: Implementation MSC: NCLEX Client Needs Category: Physiological Integrity NOT: Concepts: Sensory Perception 7. The nurse is assessing the patient's ability to hear. Which is the correct procedure for the doing this? a. The nurse whispers to the patient while standing on each side of the patient. b. The nurse speaks in a normal voice while standing on each side of the patient. c. The nurse speaks in a normal voice while standing directly in front of the patient. d. The nurse speaks in a normal voice while standing slightly behind the patient. ANS: D Hearing ability can be determined by observing the patient's conversation and responses and by talking with the patient in a normal conversational tone while standing slightly behind the patient. If the patient does not respond appropriately, a hearing impairment may exist. Standing in front of the patient allows the patient to read your lips and will not detect a hearing loss. A whispered voice will also give a false reading. DIF: Applying REF: p. 718 OBJ: 31.3 TOP: Assessment MSC: NCLEX Client Needs Category: Physiological Integrity NOT: Concepts: Sensory Perception 8. The nurse notices her 50-year-old patient is holding his lunch menu at arm's length while trying to read his choices. This is an indication of: a. retinopathy. b. presbyopia. c. cataracts. d. macular degeneration. ANS: B The patient demonstrates presbyopia by holding reading materials at a distance or by being unable to read normal-sized or small print. Retinopathy is damage to the retina and occurs in diabetics. Cataracts are a clouding of the lens. Macular degeneration is a chronic condition that causes loss of vision in the center of your field of vision. DIF: Applying REF: p. 718 OBJ: 31.3 TOP: Assessment MSC: NCLEX Client Needs Category: Physiological Integrity NOT: Concepts: Sensory Perception 9. The nurse is providing discharge education to her patient with diabetes regarding foot care. Which of the following statements by the patient indicates a need for further education? a. "I can go barefoot outside only in the summer." b. "I should wear good fitting shoes." c. "I cannot soak my feet in a hot tub." d. "I can use lotion on my feet." ANS: A Diabetic patients should not go barefoot outside even in the summer as they often have neuropathy, which decreases the patient's ability to discern touch, especially in the lower extremities. This can lead to foot injuries that can become infected and are slow to heal. The patient should wear good fitting shoes, should avoid extreme temperatures, and can use lotion to keep their skin moist to avoid overly dry skin. DIF: Applying REF: p. 718 OBJ: 31.3 TOP: Assessment MSC: NCLEX Client Needs Category: Physiological Integrity NOT: Concepts: Sensory Perception 10. An appropriate goal for a patient with the diagnosis of acute confusion is: a. the patient will use the call light before getting out of bed within 48 hours. b. the patient will use a calendar to remember the date within 48 hours. c. the patient will respond appropriately to questions about place within 48 hours. d. the patient will remain within the unit while in long-term care. ANS: C The patient has acute confusion and therefore an appropriate early goal as the confusion resolves is to remember where they are. Remembering to use a call light would be appropriate for Risk for falls. Using a calendar is appropriate for Impaired memory, and remaining in the unit is appropriate for Chronic confusion. DIF: Applying REF: p. 720 OBJ: 31.5 TOP: Assessment MSC: NCLEX Client Needs Category: Physiological Integrity NOT: Concepts: Cognitive and Sensory Perception 11. An appropriate goal for a patient with a diagnosis of social isolation is: a. the patient will participate in cognitive exercises. b. the patient will interact with other residents during activities. c. the patient will communicate basic needs through use of photos. d. the patient will remain within the unit while in long-term care. ANS: B Interacting with others during activities is an appropriate goal to help the patient not feel so alone. Cognitive exercise is a goal for a patient with disturbed thought processes. Communication of basic needs through the use of photos is a goal for a patient with a diagnosis of Impaired verbal communication, and remaining in the unit is appropriate for Chronic confusion. DIF: Applying REF: p. 720 OBJ: 31.5 TOP: Assessment MSC: NCLEX Client Needs Category: Physiological Integrity NOT: Concepts: Cognition and Sensory Perception 12. The nurse is educating the family to care for a patient at home with cognitive alterations. Which statement by the family indicates a need for further education? a. "I should keep the home free of scissors." b. "I should minimize the number of visitors." c. "I should use push-button door locks." d. "24-hour supervision may become necessary." ANS: C Use of door locks that require a key may be necessary if the patient wanders. Keep the environment free of hazards such as sharp objects, and minimize distractions. If the patient is not safe to be left alone, 24-hour supervision may be necessary. DIF: Applying REF: p. 725 OBJ: 31.6 TOP: Assessment MSC: NCLEX Client Needs Category: Physiological Integrity NOT: Concepts: Cognition 13. The nurse is delegating care to an unlicensed assistive personnel (UAP) to a patient who has sensory overload. Which statement by the UAP indicates a need for further orientation? a. "I should keep the noise levels low." b. "I should schedule all the care together." c. "I should keep the room well lit." d. "I should allow the family to visit." ANS: C To prevent or alleviate overload, the nurse reduces sensory stimuli, dimming unnecessary lights and turning down the sound on alarms if possible. Nursing care is planned so that the patient is not constantly disturbed. Visitation by family provides reality orientation and a soothing, recognizable presence for some patients experiencing overload. DIF: Applying REF: p. 726 OBJ: 31.6 TOP: Assessment MSC: NCLEX Client Needs Category: Physiological Integrity NOT: Concepts: Sensory Perception 14. The nurse is providing discharge instructions to a patient with visual alterations. Which statement by the patient indicates a need for further education? a. "I should make sure the passageways are wide." b. "I should remove all the throw rugs." c. "I should keep the lights dim." d. "I can use a cane to feel for objects in front of me." ANS: C Bright lighting in hallways and stairways prevents falls by the patient who has limited vision. Furniture is placed to allow wide passageways. Throw rugs, which are a tripping hazard, are removed. If vision is severely limited, use of a cane or walking stick held slightly in front helps the patient feel objects in his/her path. DIF: Applying REF: p. 725 OBJ: 31.6 TOP: Assessment MSC: NCLEX Client Needs Category: Physiological Integrity NOT: Concepts: Sensory Perception MULTIPLE RESPONSE 1. The nurse is completing her assessment of an older adult and notices some cognitive impairment not normally associated with aging. Which of these alterations would prompt further follow-up? (Select all that apply.) a. The patient does not remember where her son lives. b. The patient is unable to balance her checkbook. c. The patient got lost in a city she never traveled to before. d. The patient often has difficulty remembering words. e. The patient got lost going to her usual grocery store. ANS: A, B, D, E Symptoms of cognitive impairment include disorientation, loss of language skills, loss of the ability to calculate, poor judgment, and memory loss. If a patient exhibits these symptoms, further investigation is needed. Some decline in cognitive function occurs with aging, such as the ability to navigate easily in new areas. DIF: Applying REF: p. 713 OBJ: 31.2 TOP: Implementation MSC: NCLEX Client Needs Category: Physiological Integrity NOT: Concepts: Cognition 2. You are providing education to the family of a patient being discharged with dementia. Which statement by the family indicates a good level of understanding of dementia? (Select all that apply.) a. "The condition is permanent and has an acute onset." b. "Alzheimer's is the most common type of dementia." c. "The condition worsens over time." d. "I should observe for wandering behavior." e. "Agitation can be worse in the evening." ANS: B, C, D, E Dementia, which is a permanent decline in mental function, has a subtle onset. The most common type of dementia is Alzheimer's disease. Dementia is not reversible and worsens over time. Behavioral problems that arise in dementia patients include wandering, agitation, repetitive behaviors, and sundowning, or worsening of agitation and confusion in the evening. DIF: Applying REF: p. 713 OBJ: 31.2 TOP: Implementation MSC: NCLEX Client Needs Category: Physiological Integrity NOT: Concepts: Cognition 3. The nurse is caring for a patient who suffered a stroke on the right side of the brain. The nurse is careful to implement what safety measures? (Select all that apply.) a. "Make sure to put a picture board in the room to communicate with the patient." b. "Place the call light on the patient's left side." c. "Leave on a light in the bathroom at night for good visibility." d. "Place the call light on the patient's right side." e. "Make sure there are no trip hazards in the patient's room." ANS: C, D, E If the damage is on the right side of the brain, there is loss of sensation and motor function in the extremities on the left side of the body and visual-spatial problems occur. Therefore placing the call light on the side where the patient is likely to be strong is important. The patient will not necessarily have communication problems but might have visual problems, so the bathroom light is helpful. If the damage is on the left side of the brain, there is loss of sensation and motor function in the extremities on the right side of the body and problems with speech occur. DIF: Applying REF: p. 714 | p. 718 OBJ: 31.2 TOP: Implementation MSC: NCLEX Client Needs Category: Physiological Integrity NOT: Concepts: Cognition and Sensory Perception 4. The nurse is performing a health history to determine the patient's cognitive status. Which questions will be best suited to elicit the information needed? (Select all that apply.) a. "Are you able to drive to the store or do errands?" b. "Do you live with anyone?" c. "Do you ever feel sad?" d. "Are you able to smell different foods?" e. "Have you noticed any difficulty adding up numbers?" ANS: A, E Smell is a sensory function. Driving and adding numbers relates to cognitive ability. Asking about being sad helps understand mood, and asking if they live with anyone helps understand social status. DIF: Understanding REF: p. 718 OBJ: 31.3 TOP: Assessment MSC: NCLEX Client Needs Category: Physiological Integrity NOT: Concepts: Cognition and Sensory Perception 5. The nurse is performing a health history to determine the patient's sensory status. Which questions will be best suited to elicit the information needed? (Select all that apply.) a. "Do you ever lose your balance?" b. "Do you wear glasses?" c. "Do you read the newspaper?" d. "Can you feel the difference between hot and cold water?" e. "Do you wear a hearing aid?" ANS: A, B, D, E Balance, eyesight, hearing and sensation are all sensory function. Asking the patient if he/she reads the newspaper is a question to determine if he/she can read or understand what is read and is aimed at cognition. DIF: Understanding REF: p. 716 OBJ: 31.3 TOP: Assessment MSC: NCLEX Client Needs Category: Physiological Integrity NOT: Concepts: Cognition and Sensory Perception 6. The nurse is caring for a diabetic patient who has had a long history of poor glucose control. For what complications is the patient at risk? (Select all that apply.) a. Sudden loss of consciousness b. Diabetic retinopathy c. Stroke d. Peripheral neuropathy e. Memory loss ANS: B, C, D, E Long-term complications of hyperglycemia may contribute to cognitive and sensory deficits such as memory loss. They also can lead to diabetic retinopathy, peripheral neuropathy, and stroke. Loss of consciousness is usually seen with hypoglycemia. DIF: Understanding REF: p. 717 OBJ: 31.3 TOP: Assessment MSC: NCLEX Client Needs Category: Physiological Integrity NOT: Concepts: Cognition and Sensory Perception 7. The nurse is caring for a patient who is hospitalized with cognitive impairment. The following interventions will assist the patient in orientation: (Select all that apply.) a. Keep a photo of the family in the room. b. Use a clock on the wall. c. Make sure the room is kept bright and well lit. d. Avoid moving the patient from room to room. e. Have the nurse introduce himself or herself to the patient. ANS: A, B, D, E The hospitalized patient with cognitive alterations is oriented by use of a clock, a calendar, and statements about the name of the location or name of the hospital. Orientation to person, place, and time is ongoing. Staff members are always identified by name, both verbally and nonverbally (with a name tag). The patient's environment is kept as constant as possible, and moving the patient from room to room is avoided. Some familiar objects, such as a family photo, are placed near the patient if the hospital stay is longer than a few days. The environment is kept free of distractions such as loud noises and bright lights. Natural lighting to provide the patient with orientation to time of day can be accomplished by opening blinds or curtains during the day and darkening the room at night. DIF: Applying REF: p. 721 OBJ: 31.6 TOP: Implementation MSC: NCLEX Client Needs Category: Physiological Integrity NOT: Concepts: Cognition 8. The nurse is caring for a patient with expressive aphasia. Which interventions will assist the nurse in communicating with the patient? (Select all that apply.) a. Use simple phrases. b. Speak loudly. c. Use yes/no questions. d. Use a picture board. e. Be patient and unrushed. ANS: A, C, D, E If the patient has damage to the speech centers of the brain, special communication methods are used. The nurse uses simple phrases and questions and speaks clearly without shouting. If the patient has expressive aphasia, asking questions that can be answered with yes or no and using a communication board with pictures may be beneficial. Do not rush the patient and be patient. DIF: Applying REF: p. 722 OBJ: 31.6 TOP: Implementation MSC: NCLEX Client Needs Category: Physiological Integrity NOT: Concepts: Cognition and Sensory Perception 9. The nurse is caring for a patient with receptive aphasia. Which interventions will assist the nurse in communicating with the patient? (Select all that apply.) a. Use simple phrases. b. Speak softly. c. Stand in front of the patient. d. Use a picture board. e. Be patient and unrushed. ANS: A, C, D, E For patients with receptive aphasia, the nurse can point to pictures that represent basic needs such as food, drink, or use of the bathroom. In dealing with patients with either type of aphasia, the nurse is patient and unrushed. If the patient has damage to the speech centers of the brain, special communication methods are used. The nurse uses simple phrases and questions and speaks clearly without shouting, and the nurse should stand or sit somewhere that enables the patient to watch facial expressions. DIF: Applying REF: p. 722 OBJ: 31.6 TOP: Implementation MSC: NCLEX Client Needs Category: Physiological Integrity NOT: Concepts: Cognition 10. The nurse is preparing discharge instructions for a patient who has tactile alterations in his legs. Which instructions should be included? (Select all that apply.) a. Verify bath water temperature is approximately 39.5° C. b. Do not use hot or cold therapy on any extremity. c. Use sturdy shoes when walking outside or on hard surfaces. d. Report any changes in skin color on your legs to your health care provider. e. Set your water heater so that scalding is not possible. ANS: C, D, E Bath water temperature should be approximately 37.8° C (100° F), and hot and cold therapy should not be used on the affected extremities, although it can be used on other areas of the body. Sturdy shoes can prevent foot injuries when there is decreased sensation in the lower extremities. Any decrease in sensation, change in the color of the skin, or wounds are reported to the health care provider. Water heaters are set so that scalding is not possible. DIF: Applying REF: p. 725 OBJ: 31.6 TOP: Implementation MSC: NCLEX Client Needs Category: Physiological Integrity NOT: Concepts: Sensory Perception 11. The nurse is preparing discharge instructions for a patient who has equilibrium alterations. Which instructions should be included? (Select all that apply.) a. Use grab bars in the tub and/or shower at home. b. Keep rooms well lit and focus ahead when walking. c. Change positions quickly to avoid dizziness. d. Use a cane or walker for stability. e. Ride in the back seat of the car and look ahead. ANS: A, B, D The patient experiencing dizziness or vertigo exercises caution when changing positions. The patient suffering from motion sickness should ride in the front seat of the car and look far ahead through the car windshield. Keeping rooms well lit and focusing ahead when walking, using grab bars in the shower and/or tub, and using canes or walkers are all good safety measures. DIF: Applying REF: p. 725 OBJ: 31.6 TOP: Implementation MSC: NCLEX Client Needs Category: Physiological Integrity NOT: Concepts: Sensory Perception

vkjhvk


Kaugnay na mga set ng pag-aaral

Chapter 4 Accounting Sample Problems

View Set

MIDTERM 1 MIS 140 CH1 delete after

View Set

GOETHE-ZERTIFIKAT A1: Start Deutsch A1, Sprechen Teil 2 (Fragen formulieren).

View Set

301 Final Post and Pre Lecture Questions

View Set